LSAT and Law School Admissions Forum

Get expert LSAT preparation and law school admissions advice from PowerScore Test Preparation.

 Brook Miscoski
PowerScore Staff
  • PowerScore Staff
  • Posts: 418
  • Joined: Sep 13, 2018
|
#64973
wwf;

(A) doesn't question a premise. The politician states that "many would agree...," and (A) points out that those many are wrong. Thus, (A) doesn't contradict the statements in the stimulus, it simply answers a question that the stimulus left open--whether those "many people" are correct.

(B) is wrong because showing that it's insufficient to be opposed to higher taxes doesn't show that it's irrelevant or unnecessary. Let's say that opposition to higher taxes is insufficient. It could still be necessary, and that would support the politician's argument. So (B) does not weaken the stimulus.
User avatar
 Trying My Best
  • Posts: 6
  • Joined: Jan 05, 2022
|
#93342
Emily Haney-Caron wrote: Tue Jan 24, 2017 6:57 pm Hi dtodaizzle,

You're right that "many would agree" is an opinion or belief. In contrast, A presents a fact contradicting that belief; it therefore substantially weakens the argument. In contrast, B isn't quite on-point; the argument isn't saying that it is sufficient, it is only saying opposing higher taxes is necessary. So even if B is true, that just means Thompson hasn't necessarily met a sufficient condition, but if he is the only one to meet a necessary condition, then he's gonna be better than the others even if he isn't actually a good leader. Make sense?
I do not understand why you claim that opposing higher taxes is actually a necessary condition. The reasoning begins with "anyone who..." which I thought was a sufficient indicator such as "people who" or "every one who." I want to make sure I understand why "opposing higher taxes" is actually the necessary condition in the author's argument as CR is super important for this exam!
 Robert Carroll
PowerScore Staff
  • PowerScore Staff
  • Posts: 1787
  • Joined: Dec 06, 2013
|
#93389
Trying My Best,

The conditional discussion is beside the point. The argument uses the fact that many people believe something to prove that that thing is true. Anything that shows that what people think is incorrect would weaken the argument. Showing that opposition to higher taxes is not sufficient for good leadership does essentially nothing - it certainly doesn't show that there's no correlation between opposition to higher taxes and good leadership. In fact, it doesn't even show that there isn't a very strong correlation - it just shows that such opposition is not sufficient. The author wasn't ever depending on its being so - many people believed that such opposition made someone a better leader. The conditional language in answer choice (B) is incorrect not because opposition was necessary but because saying it's not sufficient doesn't hurt anything the author tried to do.

Robert Carroll
 darrengao
  • Posts: 6
  • Joined: Feb 14, 2023
|
#100624
This argument has so many holes in it ironically making it one of the hardest question I have seen. I rejected A because the argument did not make a causal claim. Yet A, saying opposing higher tax does not contribute to being a good leader denied a causal claim. That didn't sit right with me. People may believe opposing higher tax is an indicator of being a good leader. Like many people believe someone with a PHD have a higher IQ than someone who doesn't. The fact that getting a PHD doesn't contribute or increase one's IQ does not weaken this claim at all, people already know that when they made that claim.

So for A to work, we need to assume that argument is making a causal claim, a very strong one too. Rather than the indication of being a good leader, people actually believe opposing higher tax makes someone a good leader, and that influence is so strong that it overpower every other causal factor. It just doesn't look like a commonsensical assumption to me. But then again, not much of this argument makes sense. So maybe that's a fair assumption in this case.
 Rachael Wilkenfeld
PowerScore Staff
  • PowerScore Staff
  • Posts: 1358
  • Joined: Dec 15, 2011
|
#100707
Hi Darren

Let's simplify the argument a bit. It says that Thompson is the best person to lead the nation because he opposes higher tax increases, a position that many people thinks makes people a better leader than those who support higher taxes. The support offered here for the conclusion is the beliefs of many people. There's not anything else given to support that he would be the best person to lead the nation. It's completely based on the beliefs of an underdefined portion of the population.

So answer choice (A) hurts the conclusion by showing that people's beliefs are not based on a factual relationship. People may believe that something is true, but that doesn't make it actually true. People may believe that opposing taxes makes someone a better leader, but our answer choice shows us that is an incorrect belief.

Hope that helps!

Get the most out of your LSAT Prep Plus subscription.

Analyze and track your performance with our Testing and Analytics Package.